Topologia da Reta: Pontos de acumulação e conjuntos compactos

$\newcommand{\N}{\mathbb N}\newcommand{\R}{\mathbb R}\newcommand{\Z}{\mathbb Z}\newcommand{\A}{\mathcal A}$Seja $X\subseteq\R$ um conjunto e $a\in\R$. O ponto $a$ chama-se ponto de acumulação de $X$, se, para todo $\varepsilon > 0$, tem-se que  $X\cap ((a-\varepsilon,a+\varepsilon)\setminus\{a\})\neq\emptyset$. O conjunto de pontos de acumulação de $X$ é denotado por $X’$. É claro que se $a\in X’$, então $a$ é um ponto aderente a $X$, mas não é verdade que todo ponto aderente é um ponto de acumulação.

Lema. Seja $X\subseteq\R$ e $a\in\R$. As seguintes são equivalentes.

  1. $a\in X’$.
  2. $a=\lim x_n$ para uma sequência $(x_n)$ com termos $x_n\in X$ dois a dois distintos.
  3. Todo intervalo aberto que contém $a$ possui infinitos elementos de $X$.

Em particular, se $X’\neq\emptyset$, então $X$ é infinito.

Demonstração. Assuma 1. Seja $\varepsilon_1=1$ e seja $x_1\in X\cap ((a-\varepsilon_1,a+\varepsilon_1)\setminus\{a\})$. Construímos recursivamente uma sequência $(x_n)$ tal que

  1. $x_n\in X$, os termos $x_n$ são distintos dois a dois;
  2. $|a-x_n|\leq 1/n$ para todo $n$.

O termo $x_1$ foi escolhido acima. Assuma que os termos $x_1,\ldots,x_{n-1}$ foram escolhidos. Seja $0<\varepsilon_n\leq 1/n$ tal que $(a-\varepsilon_n,a+\varepsilon_n)$ não contém nenhum termo $x_1,\ldots,x_{n-1}$. Então $X\cap((a-\varepsilon_n,a+\varepsilon_n)\setminus\{a\})\neq\emptyset$. Seja $x_n$ um elemento de $X\cap((a-\varepsilon_n,a+\varepsilon_n)\setminus\{a\})$. Então $x_n\in X$, $|a-x_n|\leq 1/n$ e $x_n$ é distinto de $x_1,\ldots,x_{n-1}$. Como $|x_n-a|\leq 1/n$, $x_n\rightarrow a$ e afirmação 2. é provada.

Assuma 2. Seja $I$ um intervalo aberto que contém $a$. Então $a$ é ponto interior de $I$ e existe $\varepsilon > 0$ tal que $a\in(a-\varepsilon,a+\varepsilon)\subseteq I$. Seja $(x_n)$ uma sequência com termos em $X$ dois a dois distintos e com $\lim x_n=a$. Existe $N\in\N^+$ tal que $x_n\in (a-\varepsilon,a+\varepsilon)$ para todo $n\geq N$. Isto implica que $x_n\in I$ para todo $n\geq N$. Consequentemente, o conjunto $\{x_n,x_{n+1},\ldots\}$ é infinito e é contido em $X\cap I$. Assim, obtem-se que afirmação 3. é verdadeira.

O fato que afirmarmação 3. implica afirmação 1. é imediato da definição de ponto de acumulação. $\Box$

Lema. Se $X\subseteq\R$, então $\overline X=X\cup X’$.

Demonstração. Exercício. $\Box$

Seja $X$ um conjunto. Se $\mathcal A=\{A_i\mid i\in I\}$ é uma família de conjuntos ($I$ sendo um conjunto de índices) tal que $X\subseteq \bigcup_{i\in I} A_i$, então $\mathcal A$ é dito uma cobertura de $X$. Uma cobertura $\mathcal A$ é chamada de cobertura aberta se os conjuntos de $\A$ são abertos. Uma cobertura $\mathcal B$ é dita subcobertura de $\A$ se $\mathcal B\subseteq \A$.

Um conjunto $X\subseteq\R$ é dito compacto, se toda cobertura aberta de $X$ possui uma subcobertura finita.

Exemplo. A família $\{(a+1/n,b-1/n)\mid n\in\N^+\}$ é uma cobertura aberta do intervalo $(a,b)$  mas não possui subcobertura finita. Portanto $(a,b)$ não é compacto.

Exemplo. Seja $X\subseteq\R$ um conjunto illimitado. A família $\{(n,n+2)\mid n\in\Z\}$ é uma cobertura aberta de $X$ que não possui uma subcobertura finita. Portanto $X$ não é compacto.

Exemplo. Se $X\subseteq\R$ é finito então $X$ é compacto.

Exemplo. Seja $X=[a,b]$ um intervalo fechado e limitado. Afirmamos que $X$ é compacto. Seja $\{A_i\mid i\in I\}$ uma cobertura aberta de $X$. Seja
$$
Y=\{x\in X\mid [a,x]\mbox{ pode ser coberto por um número finito dos $A_i$}\}.
$$
Como $a\in Y$, tem-se que $Y\neq\emptyset$. Como $Y$ é limitado superiormente, existe $c=\sup Y$. Claramente, $a<c\leq b$. Afirmamos que $c\in Y$. Como $c\in X$, existe $A\in\mathcal A$ tal que $c\in A$. Além disso, $A\in\mathcal A$ é aberto, então $c$ é ponto interior de $A$ e existe $\varepsilon>0$ tal que $c-\varepsilon>a$ e o  intervalo $I=(c-\varepsilon,c+\varepsilon)$ satisfaz $c\in I\subseteq A$. Seja $x=a-\varepsilon/2$. Temos que $x\in Y$ então existe uma cobertura  $\{A_{i_1},\ldots,A_{i_k}\}$ de $[a,x]$. Neste caso   $\{A_{i_1},\ldots,A_{i_k},A\}$ é uma cobertura de $[a,c]$. Portanto $c\in Y$.

Afirmamos agora que $c=b$. Assuma que não. Seja $\{A_{i_1},\ldots,A_{i_k}\}$ uma cobertura de $[a,c]$. Assuma sem perder generalidade que $c\in A_{i_k}$. Então $c$  é ponto interior a $A_{i_k}$ que implica que existe um $\varepsilon>0$ tal que $c+\varepsilon\leq b$ e $(c-\varepsilon,c+\varepsilon)\subseteq A_{i_k}$. Isto implica que a cobertura $\{A_{i_1},\ldots,A_{i_k}\}$ é uma cobertura de $[a,c+\varepsilon/2]$ e $c+\varepsilon/2\in Y$ que contradiz ao fato que $c=\sup Y$.

Os dois exemplos mostram os dois principais impedimentos para um conjunto ser compacto. De fato provaremos que um conjunto é compacto se e somente se ele é fechado e limitado.

Teorema. As seguintes afirmações são equivalentes para um conjunto $X\subseteq \R$.

  1. $X$ é fechado e limitado.
  2. $X$ é compacto.
  3. Todo subconjunto infinito de $X$ possui um ponto de acumulação pertencente a $X$.
  4. Toda sequência $(x_n)$ tal que $x_n\in X$ possui uma subsequência $(x_{k_n})$ convergente tal que $\lim x_{k_n}\in X$.

Demonstração. 1.$\Rightarrow$ 2. Seja $X$ fechado e limitado. Portanto $A=\R\setminus X$ é aberto. Como $X$ é limitado, $X\subseteq [a,b]$ com algum intervalo fechado $[a,b]$. Assuma que $\{A_i\mid i\in I\}$ é uma cobertura aberta de $X$. Então $\{A_i\mid i\in I\}\cup\{A\}$ é uma cobertura de $[a,b]$. Pelo exemplo acima, $[a,b]$ possui uma subcobertura $\{A_{i_1},\ldots,A_{i_k}\}$ que também será uma cobertura que $X$.

2.$\Rightarrow$ 3. Seja $X$ um conjunto compacto e seja $Y$ um subconjunto que não possui ponto de acumulação em $X$. Se $x\in X$, então $x$ não é ponto de acumulação de $Y$. Portanto, existe $\varepsilon_x>0$ tal que $((x-\varepsilon_x,x+\varepsilon_x)\setminus\{x\})\cap Y=\emptyset$. Seja $I_x=(x-\varepsilon_x,x+\varepsilon_x)$. A família $\{I_x\mid x\in X\}$ é uma cobertura aberta de $X$. Como $X$ é compacto, existem $x_1,\ldots,x_k\in X$ tal que $\{I_{x_1},\ldots,I_{x_k}\}$ é uma cobertura finita de $X$. No entanto $I_{x_i}\cap Y\subseteq\{x_i\}$ vale para todo $x_i$. Isto implica que $Y\subseteq\{x_1,\ldots,x_k\}$ e que $Y$ é finito.

3.$\Rightarrow$ 4. É óbvio pelo lema acima.

4.$\Rightarrow$ 1. Assuma que  $X$ não é limitado superiormente. Então para todo $n\in \N^+$ existe $x_n\in X$ tal que $x_n\geq n$. Neste caso, $x_n\rightarrow\infty$, and so $x_n$ não possui uma subsequência convergente com limite em $X$. Logo se $X$ não é limitado, então afirmação 4. não vale para $X$.

Assuma agora que $X$ não é fechado. Neste caso existe $a\in \R\setminus X$ tal que $a$ é ponto aderente a $X$. Existe uma sequência $(x_n)$ convergente tal que $x_n\in X$ e $\lim x_n=a$. Todas as subsequências $(x_{k_n})$ de $(x_n)$ são convergentes com $\lim x_{k_n}=a$. Portanto, se $X$ não é fechado, então afirmação 4. não vale para $X$. $\Box$

Leave a Reply

Your email address will not be published. Required fields are marked *